Evidence Questions/Examples

Pataasin ang iyong marka sa homework at exams ngayon gamit ang Quizwiz!

Admit or Exclude: Defendant cop charged with rape. Prosecution wants to introduce evidence that the cop previously forced woman to perform oral sex after traffic stop

Admit

Admit or Exclude: Evidence: Complainant previously made false rape claims.

Admit

Would the following evidence be admitted or excluded in a rape case: "Aaron is so hot. I just wish I could get some time alone with him."

Admit

Viola and Antonio collide while skateboarding at the local Skate Park. Both of their skateboards are totaled, and both parties are injured. The day after the collision, Antonio gives Viola a brand new skateboard. The next week, Viola sues Antonio for negligence, and at trial she seeks to testify that Antonio gave her a new skateboard the day after the event. Antonio objects. The judge should: 1. Preclude the evidence under Rule 401, since it is irrelevant. 2. Preclude the evidence under Rule 403. 3. Preclude the evidence under Rule 408. 4. Admit the evidence.

Admit the evidence. This is unilateral, therefore not a settlement claim. Also, the evidence is unlikely to lead the jury to decide on facts other than the law of negligence, so it would not be unfairly prejudicial.

Would the following evidence be admitted or excluded in a rape case: "Aaron is so hot. I've been dreaming about sex with him."

Admitted

Which of the following may be a remedial measure? 1. Firing an employee 2. Recalling a product 3. Changing a policy 4. All of the above

All of the above

MaryAnn is suing Ginger, alleging that Ginger stole her black silk dress when they were roommates. At trial, MaryAnn calls Thurston to the stand, who testifies that he saw Ginger wearing the black silk dress in question at a party two weeks after Ginger moved out of MaryAnn's apartment. The trial ends in a mistrial, and the case is re-tried. At the re-trial, Thurston does not testify. MaryAnn seeks to admit a transcript of Thurston's testimony at the first trial. Ginger objects, and the judge overrules the objection. Ginger then calls Thurston's widow Lovey to the stand. Lovey will testify that she had been married to Thurston for forty years and that in her opinion Thurston was a dishonest man who lied about everything. MaryAnn objects to Lovey's testimony, and the judge overrules the objection. Were either of the judge's rulings correct? (1) Yes, Thurston is unavailable so his testimony is admissible as former testimony, and Lovey's live testimony should be admitted. (2) Yes, Thurston's prior testimony is admissible whether or not he is unavailable, and Lovey's live testimony should be admissible. (3) No. Thurston's prior testimony should not have been admitted no matter what. But if the judge admits it, the judge should admit Lovey's live testimony. (4) No. Thurston's prior testimony is admissible, but Lovey's live testimony should be precluded.

(1) Yes, Thurston is unavailable so his testimony is admissible as former testimony, and Lovey's live testimony should be admitted. This is a civil case so you do not need to worry about the 6th Amendment. You have an out of court statement made by Thurston at the prior trial. It is being entered for the truth of the matter that he saw Ginger wearing Mary Ann's dress. Thurston doesn't appear to be on the stand, so you cannot use the 801 exemption for prior statements. You do not know if he is unavailable for valid reasons, but assuming he is, which the first answer does, you can use the former testimony exception. This is the same parties, and Ginger had the opportunity to examine Thurston and the prior trial and the motive to do so. This gets you to the correct answer, answer 1, without needing to go any further. If, however, you are to examine the second ruling, Lovey is a character witness being called to provide an opinion about the declarant's character of dishonesty. Doing so is permissible under Rule 608, and under Rule 806, it is permissible.

Three factors to focus on when trying to determine if something is a habit: (1) __________ of the conduct (2) _________________ of the situation producing the conduct (3) _____________ of the conduct

(1) specificity (2) distinctiveness (3) regularity

To admit a prior inconsistent statement for the truth of the matter asserted (as hearsay) it must (1) have been made by a _____________ ___________ (2) who is subject to __________ ______________ (3) under _______ and (4) in a _______________ or deposition.

(1) testifying witness (2) cross-examination (3) oath (4) proceeding

The furnace in Donald's basement exploded, destroying the house and severely burning Donald. Donald sued the manufacturer of the furnace, Steamfast Furnace, for unsafe design. Steamfast's defense is that the contractor who installed the furnace five years ago did so improperly, placing the furnace in a room that was far too small. Steamfast also claims that Donald knew about the improper installation and assumed the risk of any explosion. The contractor's president, Edward, died last year and the company went out of business; neither Edward nor the company is a party to the lawsuit. At trial, Steamfast offers a letter written by Edward to Donald just before the furnace was installed. The letter says: "I have advised you not to install a Steamfast furnace because your basement is too small to handle that product. The furnace won't have adequate space to vent, and the whole thing could explode. We're violating municipal code standards by doing this. But it's your money and your house, so we'll go ahead and install the furnace you've insisted on." The letter is properly authenticated, but Donald's attorney objects to it as hearsay. The letter is: (1) Inadmissible for any purpose (2) Admissible to show knowledge on the part of Donald, but inadmissible to show that the installation was improper. (3) Admissible to show both knowledge on the part of Donald and that the installation was improper. (4) Admissible to show knowledge on the part of Donald, and admissible to prove that the installation was improper only if circumstances corroborate the trustworthiness of the letter.

(3) Admissible to show both knowledge on the part of Donald and that the installation was improper. We know from the question that we are dealing with a hearsay issue. From the answers, we can tell the first determination we need to make is whether there is a non-hearsay purpose for which the statement can be used. The assumption of risk defense requires proving Donald had notice of the situation. Showing notice is not using the statement for the truth of the matter so this means we can eliminate the first answer. The last answer suggests we should consider the opposing party defense. It seems correct that if we can prove Edward was an agent or authorized by Donald to speak, the statement would be admissible for the truth of the matter asserted (that the way the furnace was installed was in a too small area which could cause explosion) if we also had corroborating evidence. So we can eliminate the second answer. Now is the most difficult part. What about the third answer? Is there another exemption or exception that will let this in even without corroborating evidence of agency or authorization? You should go down your checklist and see statement against interest and test whether it applies here. Yes, Edward was subjecting his company to civil liability - no reasonable person who didn't want to be sued later would make this statement. It isn't a criminal case, so no corroboration is needed. This means the third answer is the most correct - the best answer. Donald hired Edwards, so Edward is Donald's agent. Steamfast wants to put the letter in as evidence against Donald. Opposing party's statement.

In the prosecution of Banana for bank robbery, a bank customer takes the witness stand and testifies, "I went down to the police station and saw a bunch of guys in, I guess, what they call a lineup, and I told them that it was that guy Banana that robbed the bank." Banana objects claiming a violation of his Sixth Amendment Confrontation Clause rights. What result? (1) The testimony is inadmissible hearsay. (2) The testimony would be admissible as a prior identification, but violates the Confrontation Clause so is inadmissible. (3) The testimony is admissible as a prior identification, and does not violate the Confrontation Clause. (4) The testimony poses no hearsay problem and is admissible.

(3) The testimony is admissible as a prior identification, and does not violate the Confrontation Clause. You have an out-of-court statement admitted for the truth of the matter, that Banana was the guy who robbed the bank. You are provided on possible hearsay exemption, prior identification. The declarant is on the stand and subject to cross so the identification falls within the exemption. The 6th Amendment is not violated by this testimony because the declarant, a bank customer, is on the stand and available for the defendant, Banana, to cross-examine. This means the third answer is the correct answer.

Chris is on trial for robbing a jewelry store. Three days before his trial is set to begin, an unknown hit-and-run driver kills the jewelry store security guard who would have been a key eyewitness against Chris. The prosecutor wishes to admit the security guard's grand jury testimony against Chris at trial. Is this permissible? (1) Yes, since the grand jury testimony was given under oath at a prior proceeding. (2) Yes, but only if the prosecutor can prove by a preponderance of the evidence that Chris was the hit-and-run driver and that he intended to make the guard unavailable. (3) Yes, but only if the prosecutor can prove by a preponderance of the evidence that (i) Chris either was the driver or acquiesced in the driver's act, and (ii) in doing so, Chris intended to make the guard unavailable for Chris' trial. (4) No, the security guard's former testimony is inadmissible hearsay.

(3) Yes, but only if the prosecutor can prove by a preponderance of the evidence that (i) Chris either was the driver or acquiesced in the driver's act, and (ii) in doing so, Chris intended to make the guard unavailable for Chris' trial. When you see the out-of-court statement of the security officer's testimony, you know you may be dealing with a hearsay issue. When you see the answers you know you have to see if the testimony fits within either the 804 former testimony exception or the forfeiture exception. The statement doesn't qualify as former testimony because there was no opportunity at the grand jury for Chris to examine the security officer. You can eliminate the first answer. Next, do either of the next two answers properly state the test to determine whether the forfeiture exception applies? Yes, the second one does. That is the proper choice.

Circumstantial Evidence

(indirect evidence) evidence used to imply a fact but not prove it directly

Who of the following would be allowed to testify as a witness at trial? 1. A party to the case who has already watched five other witnesses testify. 2. A juror who wishes to testify about alleged drug use during jury deliberations. 3. A psychotic woman who cannot distinguish between fantasy and reality. 4. The trial judge in the case, in order to advise the jurors of her opinion about the credibility of the witnesses they have heard.

1. A party to the case who has already watched five other witnesses testify.

In a landlord/tenant dispute, the landlord wishes to admit the lease, written by the landlord and signed by the landlord and the tenant, to prove the amount of rent agreed to by both parties. 1. Admissible 2. Inadmissible

1. Admissible This question also harkens back to our first lesson about the opposing party exemption. Remember that any statement adopted by the opposing party is admissible. Here our case is Landlord v. Tenant. The tenant adopted the lease by signing it. The landlord can admit it pursuant to the 801 exemption. The first choice is correct. Common example of adopting a statement → signing something that the other party drafted

Last year one of Patriot Mining's coal mines suffered a cave-in, trapping and killing three of its miners. When the bodies of the miners were discovered, there was a handwritten note found next to one of the bodies. It read: "I was placing the blasting units along the far wall and they detonated prematurely, knocking out support beam. Cave collapsed. Now it's getting hard to breathe. I'm the last one alive." The families of the dead miners are suing the manufacturers of the blasting units, claiming that the blasting units were defective. They offer the handwritten note into evidence after it is properly authenticated. The defendant corporation objects on hearsay grounds. The note is: 1. Admissible as a dying declaration 2. Admissible as a statement against interest 3. Admissible as a present sense impression 4. Inadmissible

1. Admissible as a dying declaration By the time we have read the answers, we should feel certain this is a hearsay question and we need to figure out which of three exceptions if any apply. If we take them in order we will hit on the correct one right away. This is a civil case, and the statement indicates the miner believed he faced imminent and certain death and describes the circumstances resulting in his death. If you decide to move on, you will see there is nothing against any cognizable interest in the statement. As to present sense impression, he is not merely describing what happened because he concludes that they detonated prematurely, so that key part of the description would not be admissible under the present sense impression exception.

Type of Character Evidence: Pertinent Trait of the Defendant 1. Under rule 404(a)(2), when may the Defendant offer this evidence? 2. Under rule 404(a)(2), when may the Prosecution offer this evidence?

1. Any time 2. To rebut character evidence of the same trait offered by the Defendant OR to match character evidence that the Defendant offered about the alleged victim

Type of Character Trait: Other Pertinent Trait of Alleged Victim (not trait of peacefulness) 1. Under rule 404(a)(2), when may a Defendant offer this evidence? 2. Under 404(a)(2), when may the Prosecution offer this evidence?

1. Any time, unless barred by Rule 412 (the rape shield law) 2. Only to rebut evidence of the same trait offered by the defendant; evidence must comply with Rule 412 (the rape shield law)

Under 608(b), the judge is most likely to allow cross-examination about the fact that the witness... 1. Cheated on an exam 2. Burned neighbors patio furniture 3. Often drunk on weekends 4. Numerous tickets for running red lights

1. Cheated on an exam

What 4 instances can you get character evidence in?

1. Child Custody Cases 2. Negligent entrustment 3. Entrapment 4. Libel/defamation

Todd was driving his van when a cow seemingly fell out of the sky and damaged his front hood. The day before, the cow was competing in the Prairie Fun League Livestock Exhibit. The head wrangler wrote a letter to Callie, the cow owner. It read "I've looked into the falling cow accident, and our cow was blameless. She was the healthiest cow in the herd. It appears that a hurricane-level wind blew out of nowhere and pushed the poor cow right off the cliff. The poor cow!!!!" The wrangler's memo is ... 1. Inadmissible hearsay 2. Admissible as a business record 3. Admissible as an excited utterance 4. Admissible as a present sense impression

1. Inadmissible Hearsay

Type of Character Evidence: Trait of Peacefulness of the Alleged Victim 1. Under rule 404(a)(2), when may a Defendant offer this evidence? 2. Under 404(a)(2), when may the Prosecuion offer this evidence?

1. Not applicable: Defendant would not introduce this evidence 2. In a homicide case: to rebut any evidence that the alleged victim was the first aggressor. In other cases: To rebut character evidence that the victim was not peaceful.

•11-year-old girl (JK) had unusual abrasions in genital area •Mother took JK to pediatrician •Mother said to pediatrician: JK told me that my live-in boyfriend (Carpitcher) hurt her there •JK gave sworn police statement: Carpitcher molested me At trial, JK recants. Can the prosecutor introduce JK's sworn police statement? 1. Only to impeach 2. To impeach and as witness's prior inconsistent statement 3. To impeach and as recorded recollection 4. No

1. Only to impeach

Refreshing Recollection: Steps 1. Witness says she can't ________ 2. Identify "_________ _________" and ask witness if that might refresh 3. Show memory jogger to ______________ __________ 4. Ask permission to _____________ 5. Show memory jogger to __________ 6. Take __________ ___________ away 7. Ask witness: "_____ ______ _________ ______ _____________?" 8. Ask witness to_________ ______ __________.

1. recall 2. "memory jogger" 3. opposing counsel 4. approach 5. witness 6. memory jogger 7. did that refresh your memory? 8. testify from memory

Pam is contesting her father's will, and on direct she testifies that at the time he signed the will, his mental condition had deteriorated to the point where he always seemed confused and could only mumble unintelligibly. On cross-examination, opposing counsel asks her: "Isn't it true that the day after the will was signed, you were talking to your friend Marge, and you said: 'I can't believe Daddy's mind is so sharp at his age!'" Pam's attorney objects, and is overruled. Pam denies that she said this. During his own case-in-chief, the defendant's attorney calls Marge to testify to the conversation. Pam's attorney objects to that as well. How would you rule on these two objections? 1. Overrule both objections--allow the attorney to ask Pam the question and allow Marge to testify. 2. Allow the attorney to ask Pam about the prior statement, but do not allow him to call Marge to the stand. 3. Allow the attorney to call Marge to the stand, but do not allow him to ask Pam about the prior statement. 4. Sustain both objections--preclude the question and Marge's testimony.

1. Overrule both objections--allow the attorney to ask Pam the question and allow Marge to testify. This question requires you to work through each objection one by one. The first objection is to the leading question asked on cross about a conversation with Marge. The statement made to Marge that her father's mind was sharp is inconsistent with the statement Pam made during direct exam that her father's mental condition had deteriorated. So you are dealing with impeachment by prior inconsistent statement. It is appropriate to ask on cross about a prior inconsistent statement so the judge correctly overruled the first objection. Next Marge is called as a witness to testify to the prior inconsistent statement. While Marge's testimony is extrinsic evidence (not testimony by the witness on the stand who is Pam), it is permissible because this is a noncollateral matter. It is crucial to the cause of action to determine the father's mental condition at the time he signed the will. The judge should permit Marge to testify. This means answer 1 is correct.

Business Record 1. ________ made at or near time of event 2. By someone with ___________ 3. Course of ____________ ____________ business 4. __________ practice

1. Record 2. knowledge 3. regularly conducted 4. regular

When refreshment occurs before testifying, the adverse party may examine the materials . . . 1. When the interest of justice require 2. Always 3. Never 4. If the witness is hostiles

1. When the interests of justice require

U.S. v. Brooks • Brooks charged with selling cocaine • Police Officer Christie Hoyle testified she bought cocaine from Brooks • Hung jury • Hoyle killed by boyfriend • New trial of Brooks Admit Hoyle's testimony from first trial under 804(b)(1)? 1. Yes 2. Only if Brooks actually cross-examined her 3. Only if probative value outweighs prejudice 4. No

1. Yes

U.S. v. David Theiss. Charged with distribution of controlled substance. •Government claims David rented hallucinogenic toad to friend Jerry. •Jerry claims Fifth Amendment privilege •Jerry's doctor testifies: Jerry said he was "coming down from a bad high," and that he had "taken some toad." Admissible as Statement for Medical Diagnosis? 1. Yes 2. No, not pertinent to treatment 3. No, it's a statement of blame 4. No, it's not trustworthy

1. Yes

U.S. v. Ricketson • Ricketson charged with robbing jewelry from elderly widow's home • Widow too ill to attend court • Government deposed widow in her home, with notice to Ricketson that she would not attend trial Widow's deposition admissible against Ricketson under 804(b)(1)? 1. Yes 2. No, depositions not admissible in criminal cases 3. No, because she is still alive 4. No, because motives at deposition differ

1. Yes

•The Consumer Product Safety Commission determined that the Midwest Fireworks Manufacturing Company sold more than seventy different types of fireworks that violated CPSC safety standards. When the company refused to comply with a "letter of advice" asking it to cease selling these fireworks, the Commission sought a civil injunction against further sale of the dangerous fireworks. At trial, the Commission offered reports from its own laboratory finding that the challenged fireworks contained more pyrotechnic powder than federal regulations allow. Are the facts and conclusions in CPSC's report admissible as public records? 1. Yes 2. Probably not

1. Yes

Southern Ohio. 16 year old girl, Cherly Fossyl, disappeared in 1997. Body found several weeks later. Police did not solve crime. Some believed they participated in a cover up. In 2005, Jean Ann Chinn was dying of cancer in Florida. She told her brother she had witnessed the murder, and named 2 men. Chinn died two months later. Grand jury - not enough evidence for criminal prosecution. Fossyl's siblings filed civil suit against alleged murderers and county. County settled. 2007 trial. Admit evidence that Chinn was using narcotic pain relievers? 1. Yes, it affects Chinn's credibility 2. No, because Chinn is not a witness 3. No, because it's unfairly prejudicial 4. No, because it's extrinsic evidence

1. Yes, it affects Chinn's credibility

A prosecutor wants to admit a written confession by the defendant into evidence to prove the defendant's guilt. 1. admissible 2. inadmissible

1. admissible This question harkens back to our original lesson about the exemption for opposing party statements. The case is Prosecutor v. Defendant. The prosecutor can admit any statement the defendant made, including the confession. So the proper choice is the first answer.

3 Ways to Show Untruthful Character

1. cross-examine about untruthful acts (608(b)) 2. Introduce evidence of prior conviction (609) 3. Offer reputation or opinion testimony about untruthful character (608(a))

Steps to Examining a Character Witness 1. Lay the ___________ 2. Ask for the witness's ____________

1. foundation (i.e., establish the character witness knows the fact witness well-enough to have formed an opinion as to their character/reputation) 2. opinion (i.e., what is your opinion of Mr. X's truthfulness?)

The Hearsay "Four" 1. Is the witness referring to a statement that ... 2. Are you sure it is a _________? 3. Is a party offering the statement to prove the ... 4. Does an ____________ apply?

1. occurred outside the courtoom 2. statement 3. truth of the matter asserted 4. exception

Recorded Recollection 1. Memorandum or _________ 2. Once had ____________ of matter it concerns 3. Now have no ______________ 4. Record made or adopted by ____________ 5. When it was _________ in memory 6. And accurately reflected that _________________

1. record 2. knowledge 3. recollection 4. witness 5. fresh 6. knowledge

The jury may use a writing introduced under Rule 612... 1. To assess credibility 2. To establish a controverted fact 3. To establish liability 4. For any purpose

1. to assess credibility

Statement Against Interest 1. Declarant ___________ 2. Statement was __________ ___________ 3. At the _______ it was made 4. ________________ for exculpatory statements when offered in criminal case

1. unavailable 2. against interest 3. time 4. corroboration

Marcia Brady is on trial for killing her stepfather Michael. In its case-in-chief, the prosecutor introduces a properly authenticated letter from Jan, Marcia's sister. The letter was found in Marcia's possession when she was arrested, and it is dated the day before the murder. In the letter, Jan wrote that Michael had physically assaulted her multiple times, including breaking her nose, over the past few weeks. The defense objects to this document as hearsay and inadmissible character evidence. The court should rule that the letter is: 1. Admissible for any purpose 2. Admissible to prove Marcia's motive for committing the crime 3. Admissible to prove that Michael actually did assault Jan 4. Inadmissible for any purpose

2. Admissible to prove Marcia's motive for committing the crime For this question you see a written document, so should be concerned you may have a hearsay issue. You also see the objections are hearsay, and our old favorite, character evidence. First address the hearsay issue. The letter says Michael assaulted Jan - so the truth of the matter is that Michael assaulted Jan. We can't use it for a hearsay purpose so can eliminate choice 3. Next ask why is the prosecutor offering the letter. The prosecutor isn't trying to prove that Michael assaulted Jan - only that Marcia thought/believed Michael had assaulted Jan so had a motive to kill Michael. So choice 2 solves the hearsay problem. Then turn to the character evidence problem. Ask is the prosecutor trying to prove that because Michael assaulted Jan previously, he is violent, and assaulted Jan or someone else in an incident at issue. No, again the prosecution is using the letter for a non-propensity purpose, to show motive, so again the evidence is permissible and choice 2 is correct.

Government charges Joseph with insider trading. Joseph's girlfriend, not wife, testifies before grand jury. At trial, the girlfriend now "forgets." The grand jury testimony is: 1. Admissible under 804(b)(1) 2. Admissible under 801(d)(1)(A) 3. Admissible under both 4. Admissible under neither

2. Admissible under 801(d)(1)(A) Girlfriend's Grand Jury Statement Is NOT Admissible as Former Testimony. • She is unavailable (memory loss) • Previous statement was at a hearing • But opponent (Joseph) was not present and had no opportunity to develop testimony But the Grand Jury Statement IS Admissible as a Prior Inconsistent Statement. • Memory loss is inconsistent with prior detailed statement • Prior statement was made under oath at a proceeding • Girlfriend IS subject to cross-examination in current proceeding

Sam is charged with raping his girlfriend Vicky. In his case in chief, he calls his friend Tom, who testifies that the day after the alleged rape, Sam told him "Vicky and I went all the way last night; I wasn't sure if we were ready, but she wouldn't take no for an answer." Sam offers the statement to prove consent. 1. Admissible 2. Inadmissible

2. Inadmissible Your first inclination here might be to think about the rape shield and whether it would prohibit this statement. The rape shield doesn't prohibit it because it isn't evidence of other sexual behavior or sexual predisposition (other acts or character), and even if it did the rape shield permits evidence of sexual behavior with the defendant to prove consent. You need to continue down your outline/checklist and realize that this actually raises a hearsay issue (not propensity or rape shield issue). This is Sam's out-of-court statement to Tom. It is a statement, and is being offered for the truth of the matter - that Vicky agreed to have intercourse with Sam. There is no exception or exemption that applies. The statement is Sam's own statement, not that of an opposing party. The correct answer is that Sam's out of court statement is inadmissible because it is prohibited hearsay

Pam is contesting her father's will. She testifies that her father was not in his right mind when he signed a new will that left all of his money to her stepmother. On cross-examination, the defendant's attorney holds up a copy of the resume she used to get her current job and asks if everything on it is true. Pam states that it is. The attorney then seeks to call Pam's current employer, who will testify that Pam claimed to have graduated from college on her resume, and she later admitted that she never in fact graduated. Pam's attorney objects. The testimony is: 1. Admissible, since it proves that Pam lied in the past and is therefore more likely to be lying now. 2. Inadmissible, because this is extrinsic evidence about a prior dishonest act. 3. Inadmissible, because it is irrelevant to her credibility. 4. Inadmissible, because Pam's employer does not have firsthand knowledge of whether Pam graduated from college.

2. Inadmissible, because this is extrinsic evidence about a prior dishonest act.

The government prosecuted Christopher Mackey for robbing four banks. Among other pieces of evidence, the government showed that Mackey spent more than $90,000 in the months following the robberies—even though he did not hold a steady job. In defense, Mackey claimed that the money stemmed from a series of successful bets on baseball games. During an FBI interview, a bookie stated that Mackey had enjoyed an "unbelievable winning streak" with his baseball bets and had won more than $60,000. The bookie refused to testify at Mackey's trial, claiming the Fifth Amendment privilege against self-incrimination. Mackey then offered an FBI report recording the bookie's statement. Is the bookie's statement recorded in the FBI report admissible as a public record? 1. Yes 2. No

2. No

U.S. v. Zapata • Alleged conspiracy to distribute drugs • Londono was conspiracy leader • Londono pled guilty; refused to testify at Zapata's trial • Zapata offers statement from Londono's plea allocution: "I never told Zapata the purpose of our enterprise" Is statement from plea allocution admissible under 804(b)(1)? 1. Yes 2. No

2. No

•Sarah and her brother work for family business •Millions stolen from business •Brother charged with crime Sarah unalives herself, leaving behind a note that reads: "I can no longer live with what I have done. I have stolen millions from the family business, and my brother has been falsely accused. Now the entire family faces financial ruin because of me. The only way out is to kill myself." Can Sarah's brother admit the note as a dying declaration? 1. Yes 2. No

2. No

•Sarah and her brother work for family business •Millions stolen from business •Brother charged with crime Sarah unalives herself, leaving behind a note that reads: "I can no longer live with what I have done. I have stolen millions from the family business, and my brother has been falsely accused. Now the entire family faces financial ruin because of me. The only way out is to kill myself." Can Sarah's brother admit the note as a statement against interest? 1. Yes 2. No

2. No

The U.S. prosecuted 18 defendants in drug conspiracy. Jack, a member of the alleged conspiracy, testified against his partners in crime. He was a self-confessed cocaine addict. On the stand, Jack acknowledged faulty memory. His testimony was inconsistent in numerous respects. Defendants moved to exclude under 602, claiming his addiction and memory loss deprived him of personal knowledge. Does Rule 602 bar Jack's testimony? 1. Yes, he lacks personal knowledge. 2. No, he has sufficient personal knowledge to testify.

2. No, he has sufficient personal knowledge to testify. •The District Judge in this case, characterized Jack as a "loose cannon" and noted his testimony may have been unbelievable to some and his perception might sometimes have been impaired. •But Jack had personal knowledge, and the opposing counsel could undermine his ability to perceive and his credibility on cross-examination.

Sally crashed into Lucy's car, and Lucy claimed Sally was negligent and sued Sally for damages. At trial, Lucy wishes to call Sally's brother Charlie, who will testify that Sally "is always speeding, and never pays attention to the road when driving." Sally objects. The court should: 1. Preclude this testimony as irrelevant 2. Preclude this testimony under Rule 404(a) 3. Admit this testimony under Rule 406 4. Admit this testimony under Rule 404(b) with a limiting instruction that it is not to be considered to prove propensity.

2. Preclude this testimony under Rule 404(a) This is a negligence claim. It is not a type of case where character is at issue. The testimony is relevant only because of the propensity logic. Sally is an inattentive driver, so she drives carelessly, so she was negligent/careless during this accident. Rule 404 prohibit precisely using this evidence for this purpose.

Sally crashed into Lucy's car, and Lucy claimed Sally was negligent and sued Sally for damages. At trial, Lucy wishes to call Sally's brother Charlie, who will testify that Sally "is always speeding, and never pays attention to the road when driving." Sally objects. The court should: 1. Preclude this testimony as irrelevant. 2. Preclude this testimony under Rule 404(a) as propensity evidence. 3. Admit this testimony under Rule 406 because character is an element of the case. 4. Admit this testimony under Rule 404(b) with a limiting instruction that it is not to be considered to prove propensity.

2. Preclude this testimony under Rule 404(a) as propensity evidence. This is a negligence claim. It is not a type of case where character is at issue. The testimony is relevant only because of the propensity logic. Sally is an inattentive driver, so she drives carelessly, so she was negligent/careless during this accident. Rule 404 prohibits precisely using this evidence for this purpose.

The United States prosecuted the mayor and several town council members of Cicero, Illinois, charging that they defrauded the town of more than $10 million. At trial, the defendants offered minutes from a town council meeting held several months after the fraud became public. According to these minutes, no payments had been made to the organization accused of perpetrating the fraud. The mayor presided over this meeting, and several other defendants were present as council members. Are the town council meeting minutes admissible as public records? 1. Yes 2. Probably not

2. Probably not

BigIndustry Inc. is being sued by several landowners in rural Ohio for illegally dumping garbage on their property. In their case-in-chief, the landowners call Walt, an inspector for the state environmental agency. He testifies that he investigated BigIndustry's practices as part of his duties as an inspector and filed a report with the agency based on his investigations. He testifies to his findings, which were that several tons of waste traceable to BigIndustry were found on the properties in question. The plaintiffs then show him a copy of the report and he verifies that it is the report that he filed after his investigation. The plaintiffs then offer the report into evidence, and the defendant objects. What is the proper ruling? 1. The report can be read to the jury as a recorded recollection, but not admitted into evidence. 2. The report is admissible as a public record. 3. The report is both a recorded recollection AND a public record. 4. The report is inadmissible because the witness has testified live on the stand as to its contents.

2. The report is admissible as a public record. For this one you can easily eliminate the last choice because you know that whether someone testifies about a document does not affect whether it is admissible. You see the other choices are asking about two hearsay exceptions, recorded recollection and public record. For recorded recollection as you start through the requirements, you will recall that the primary requirement is that the witness no longer recalls the information that was previously memorialized. So you can eliminate the first and third answers. That leaves the second choice. Another approach is to apply all the requirements of the public record exception, which are sometimes difficult to recall, but you can put them on your cheat sheet for the exam. This is a government employee, so you must use public records and not the business records exception. The report seems to fit best in the third type of report, from a legally authorized investigation. It is a civil case so it is admissible, and there is no indication it is untrustworthy

Gumbel has sued Alvert for breach of contract. Gumbel claims he purchased an order for 1,000 basketballs, and they were never delivered. Alvert claims that he delivered them on time and that Gumbel is fraudulently claiming he never received them. At trial, Alvert testifies that one week before the order was due, he told his partner Costas "I'm going to ship the Gumbel order tomorrow." Gumbel objects to the statement as hearsay. 1. The statement is not hearsay because the declarant is now testifying on the stand and can be cross-examined as to the statement. 2. The statement is hearsay but is admissible under 803(3) as state of mind. 3. The statement is admissible as a party-opponent admission. 4. The statement is inadmissible hearsay.

2. The statement is hearsay but is admissible under 803(3) as state of mind. We have an out-of-court statement that Alvert offers for the truth of the matter, that he planned to ship the shipment. He wants to use it as circumstantial evidence that he acted consistent with his plan and did ship the order. It is something he is going to do -- in other words a plan. Plans are completely admissible as to the planner, and so 803(3) lets this in as evidence of Alvert's state of mind.

Cross-examination is limited to ... 1. The subject matter of direct examination 2. The subject of direct exam and credibility 3. Credibility only 4. Any matter of consequence to the action

2. The subject of direct exam and credibility

Maria sued her employer for overtime violations because she was not paid for time spent removing her safety gear after her shift. During settlement negotiations, her employer agreed that it owed her overtime payments. The employer explained that Maria took twice as long as other employee s to remove her gear so they were not willing to pay her for the full amount of hours of overtime she demanded. Maria and her employer settled with the employer agreeing to pay Maria for half of the hours she claimed she had worked removing her safety gear. A year later the Kentucky Labor Cabinet sued Maria's employer for overtime violations for failing to pay ten other employees for time spent removing their safety gear. The attorney for the Kentucky Labor Cabinet would like to call Maria at the trial to testify to her employer's statement during settlement negotiations that it owed her overtime payments. The attorney: 1. Will be able to call Maria to testify to the admission during the settlement conference to prove her employer's liability for failure to pay overtime wages because the government is involved. 2. Will not be able to call Maria to testify to the admission during the settlement conference to prove her employer's liability for failure to pay overtime wages. 3. Will only be able to call Maria to testify to the admission during the settlement conference to prove her employer's liability for failure to pay overtime wages if the Labor Cabinet is prosecuting a criminal case. 4. Will only be able to call Maria to testify to the admission during the settlement conference to prove her employer's liability for failure to pay overtime wages if the Labor Cabinet is bringing a civil case

2. Will not be able to call Maria to testify to the admission during the settlement conference to prove her employer's liability for failure to pay overtime wages. Rule 408 governs statements made during settlement negotiations in a civil trial. The Rule prohibits using the statements to prove liability. The Rule applies equally whether the statements are being offered in a civil or criminal case. The only narrow exception, when a government agency first brings a civil case in which the statement is made during settlement negotiations and then the same or another government agency brings a criminal case and is able to introduce the statement, does not apply to these facts. The initial suit here was brought by an individual, Maria, and not a government agency. Thus the correct answer is 2 - which states the generally applicable rule.

Meredith told her friend, Tess, about her new job. Meredith said, "Last week, I accepted a job with the U.S. Attorney's Office." Can Tess testify about Meredith's statement to show she was in a positive mood? 1. Yes, state of mind. 2. Yes, not hearsay. 3. Yes, excited utterance. 4. No, hearsay.

2. Yes, not hearsay.

Who may introduce into evidence a writing used to refresh recollection? 1. Any party 2. An adverse party 3. The witness 4. No one

2. an adverse party

Under the FRE, state law determines competence ... 1. in criminal misdemeanor cases 2. in civil cases, on elements for which state law provides rule 3. bankruptcy cases 4. in civil cases worth less than $25,000

2. in civil cases, on elements for which state law provides rule

Steve Austin, a gold-medal sprinter in the 2008 Olympic Games, is now on trial for illegally possessing performance-enhancing steroids in 2007. To prove the requisite intent, the prosecution must show Austin knew he possessed steroids. The prosecution's star witness is Austin's trainer, Dr. Wells, who testifies that he sold Austin thousands of dollars worth of Detrimin, an illegal steroid, in 2007. Austin took the stand in his own defense and admitted that he bought and used the Detrimin but claimed that he did not know what it was; that he trusted Dr. Wells and that Dr. Wells told him Detrimin was a nutritional supplement. In rebuttal, the prosecution calls Oscar Goldman, an old friend of Austin's who testifies that while the two men were training for the 2004 Olympics together, Austin showed Goldman a pill bottle marked 'Detrimin' and said: 'These are some steroids I've started using. You should try some; they've really improved my training regimen.' Austin has never been charged nor convicted of using steroids in the year 2004. Goldman's testimony is: 1. Inadmissible hearsay 2. Not barred under the hearsay rule but still inadmissible for any purpose 3. Admissible to prove that Austin had knowledge that he was taking steroids in 2007 4. Admissible to prove Austin has a propensity to use steroids when training.

3. Admissible to prove that Austin had knowledge that he was taking steroids in 2007 First, you notice that the witness Goldman testifies Austin "said." When you hear the word "said" that is an indicator you may have a hearsay issue. Looking at the choices you can eliminate 4 because we have already learned that 404 prohibits propensity evidence, unless an exception applies. That leaves you with three choices. Check whether the statement is being used for a hearsay purpose. The statement's content is that Detrimin is a steroid that Austin is using. The prosecution is trying to prove that Austin knew Detrimin was a steroid, not that Detrimin is a steroid, which is an otherwise verifiable fact, or that Austin was using steroids in 2004 (they need proof he was using Detrimin in 2007). Because they are trying to prove knowledge and not the truth of the matter asserted in Austin's out-of-court statement, they can admit the testimony for that non-hearsay purpose. (Note that generally opposing party statements are admissible even for hearsay purposes as we will later learn, but that is not an answer choice here.)

Is the following admissible? Psychologist: "Christine told me that the stranger grabbed her, twisted her around, and flung her several feet in the air." 1. Admissible, present sense impression. 2. Admissible, excited utterance. 3. Admissible, medical treatment. 4. Not admissible, hearsay.

3. Admissible, medical treatment.

Ari Fleischer, the former White House Spokesman, is testifying for the prosecution against Scooter Libby. On direct, Fleischer testifies that on July 11, 2003, Libby told him about the identity of a CIA agent and told him to give the information to reporters. On cross-examination, Libby's attorney notes that Fleischer had been under investigation by the special prosecutor beginning in June of 2004, but that in December 2004 he was told he was not going to be indicted. Libby's attorney asks Fleischer if he changed his story to implicate Libby in order to avoid being charged with a crime himself. Fleischer denies that he ever changed his story. On re-direct, the prosecutor offers into evidence notes which Fleischer himself had made on July 11, 2003, which stated "Scooter told me about CIA operative today & said I should tell reporters." The defense attorney objects to this statement as hearsay. 1. The statement can be admitted, but only to prove that Fleischer is credible, not for the truth of the matter asserted. 2. The statement is admissible as a party-opponent admission, since Fleischer and Libby were co-conspirators. 3. The statement is admissible under 801(d)(1) as a prior consistent statement for any purpose. 4. The statement is inadmissible for any purpose.

3. The statement is admissible under 801(d)(1) as a prior consistent statement for any purpose. First notice that the writing is an out of court statement. It is a statement - an assertion or communication - by Fleishmaker to himself or others who might read the notes. It was made out of court, wherever Fleisher was on July 11, 2003. Remember, even a witnesses own statements made out-of-court can constitute hearsay. You, the attorney, have to ask a witness what they perceived not what they said. The truth of the statement is that Scooter told Fleishmaker about a CIA operative on July 11, 2003, and Scooter told Fleishmaker to tell reporters. While Scooter's statement to tell reporters is likely not a statement because it is a direction, similar to the Capitan saying blow your horn three times, Fleishmaker's statement that Scooter did tell him that is ascertainable of truth or falsity - Scooter did or did not tell him to tell reporters. Is the attorney using the statement to prove that Scooter told Fleishmaker these things on July 11, 2003. This is debatable, either the prosecutor is using it for the truth of the matter or is simply using it to butress Fleishmaker's credibility. Whichever one it is, always move to the next step and look for an exemption or exception so you can get the statement in without a limiting instruction. We can eliminate the second answer because the evidence is his own statement not that of an opposing party. That leaves us to determine if the notes are admissible as a prior consistent statement. Fleisher is on the stand and subject to oath and cross. Opposing counsel has asserted he had a reason as of June 2004 to lie about what Libby told him so that his own investigation would not result in charges. July 11, 2003 is before June 2004, so the notes suggest he did not change his story. Thus, 801(d)(1) permits the notes to be admitted for the truth of the matter asserted.

An attorney may refresh recollection ... 1. Only on direct examination 2. Whenever the recollection is relevant 3. Whenever the judge grants permission 4. Only on cross-examination

3. Whenever the judge grants permission

What is a fact witness?

A fact witness testifies about the facts related to the underlying legal dispute

Daniel Osborne, a former guitarist in the band Dingoes Ate My Baby ("DAMB") has recently launched a successful solo career. His first album: "Werewolves in California," has sold over one million copies on the Capitol Records label. Unfortunately the remaining members of DAMB, Osborne's former band, are suing Osborne. They claim that the lyrics to the title track of his new album were stolen from a DAMB song of the same name from four years ago. At trial, the lead singer for DAMB presents handwritten lyrics which he claims to have written four years ago and which are identical to the lyrics to "Werewolves in California." Later in the plaintiffs' case, in order to prove damages, the plaintiffs present the jury with sales records obtained from Capitol Records that purport to show that "Werewolves in California" sold 1.2 million copies. Osborne's attorney objects to the admissibility of the handwritten lyrics and to the admissibility of the sales figures from Capital Records. The judge should: 1. Preclude the lyrics and the sales figures, since both are hearsay. 2. Preclude the lyrics as hearsay but admit the sales figures. 3. Admit the lyrics and the sales figures as business records if certified by a qualified representative. 4. Admit the lyrics but preclude the sales figures as hearsay

3. Admit the lyrics and the sales figures as business records if certified by a qualified representative. Once you read the answers, you know you are dealing again with a question that is asking about whether a hearsay exception applies. While there is certainly an argument the lyrics are not being used to prove the truth of the matter - no one cares if what happens in the song is true or false - that is not one of your choices. Remember, always move on to see if an exception applies, even for a statement that is not being used for a hearsay purpose - that is the key to this question. Both documents are business records. The songwriter is in the songwriting business and ordinarily and regularly writes songs as part of that business. When the songwriter writes the lyrics down, they are fresh in his mind, and nothing indicates untrustworthiness. The same requirements are satisfied by the sales figures. The record company keeps records of sales regularly in the course of its business. The person who records them or from whom the recorder receives them has knowledge of their correctness at the time they are recorded. Nothing indicates they are untrustworthy.

Ted is on trial for killing Steve while both men were watching their sons play football. Ted testifies in his own defense, and says that he and Steve were arguing about a ruling on the field when Steve suddenly came at his throat with a knife. Ted calls no character witnesses during his case. In rebuttal, the prosecutor calls Helen, Steve's wife, to testify, over the defendant's objection, that she knew Steve for twenty-two years and that he was "peaceful and non-violent." On cross-examination, the defense attorney asks Helen if she knew that Steve had been arrested for assaulting a man with a knife nine years ago. What are the proper evidentiary rulings? 1. Allow Helen's direct testimony under Rule 404, but preclude the question on cross-examination under Rule 405. 2. Allow Helen's direct testimony under Rule 404, but only if she also testifies as to specific instances of conduct on the part of Steve to support her opinion. The question on cross-examination should be permitted under Rule 405. 3. Allow Helen's direct testimony under Rule 404, and allow the question on cross-examination under Rule 405. 4. Preclude Helen's testimony entirely, based on Rule 404

3. Allow Helen's direct testimony under Rule 404, and allow the question on cross-examination under Rule 405. The first thing to spot is that Ted killed Steve so this is a homicide trial! The second thing to spot is that this is character evidence that the prosecution is using to make a propensity argument that Steve was peaceful and would not have come at Ted with a knife. Then think about if any exception will permit the prosecution to make such an argument. You should hit on the criminal case exception which permits the prosecution to present evidence of a victim's character for peacefulness in a homicide case if the defense puts on evidence the victim was the first aggressor. Ted did that when he testified that Steve came at his throat with a knife. This means the prosecution's witness is admissible. From that you know if the character witness testifies, then the defense can ask about specifics on cross-exam, so both objections should be overruled making 3. the correct answer. Another approach would be to rule out answer 2 because you know that specific instances cannot be inquired into on direct so it must be wrong. Then rule out answer 1 because you know that if Helen's allowed to testify then questions about specific acts on cross would be permitted. That leaves you with answers 3 & 4 - a 50% chance!

Hollywood "Bling Ring" •Nick Prugo, Rachel Lee, and four others have been charged with burglary •Suppose Lee testified to grand jury, incriminating others •But at trial, she takes the 5th am and refuses to testify Can the prosecutor introduce Lee's grand jury testimony against others in the "ring"? 1. Yes, she's a co-conspirator 2. Yes, this is a prior statement and Lee is "available" 3. No, this violates the Sixth Amendment

3. No, this violates the Sixth Amendment

George was driving down Highway 33 when a herd of cattle stampeded across the road in front of him. He struck one of the cows and totaled his car. He then sued the state for maintaining an unsafe road. Two days later, the state erects a number of signs along the road, warning drivers of the cow crossing. Two weeks after that, the landowner (who is a not a party to the case) puts up a fence along the road to prevent the cows from wandering into traffic. At trial, the state expert testifies that although the state has an easement on the land running alongside the road which allows the state to place signs on the property, it has no legal right to erect a barrier of any kind. The state expert concedes on cross-examination that warning signs might make the road slightly safer, but argues that the road is adequately safe without signs and points out that there were no prior accidents involving cars striking cows on the highway. On rebuttal, the plaintiff's attorney seeks to admit evidence of the signs and the fence erected after the accident. The judge should: 1. Preclude evidence of both 2. Admit evidence of both 3. Preclude evidence of signs, but perhaps admit evidence of fence. 4. Preclude evidence of fence, but perhaps admit evidence of signs.

3. Preclude evidence of signs, but perhaps admit evidence of fence. This is a difficult one because you can't eliminate chooses. Instead you have to analyze each piece of evidence. Starting with the signs, they are subsequent and they are a remedial action - designed to make the road safer. The state has not contested feasibility but has argued only that it was adequately safe without signs. That leaves the only purpose for the evidence to be to prove liability, which Rule 407 prohibits. Thus the answer has to be either 1 or 3. The fence is a subsequent remedial measure, but it was taken by someone who isn't a party to the case. Generally judges let these in, though some may not. Remember, rule 407 applies to parties in the cases.

Katrina lives near wilderness, and wildfires have broken out. Katrina says, "I'm scared. The fires have been raging for weeks." What does the sentence "I'm scared" fall under? 1. Excited Utterance 2. Present sense impression 3. State of mind 4. None of these

3. State of mind

Charles Dawson is on trial for murder. He wishes to testify about his alibi, but he has two prior criminal convictions: (1) a misdemeanor conviction for filing a false police report fifteen years ago; and (2) a felony drug sale conviction from eleven years ago. Dawson spent three years in prison for the drug sale conviction. Which of these (if either) are admissible against him? 1. Both crimes are admissible only if their probative value substantially outweighs their unfair prejudicial effect. 2. The false police report is automatically admissible; the drug sale is admissible only if its probative value substantially outweighs its unfair prejudicial effect. 3. The false police report is admissible only if its probative value substantially outweighs its unfair prejudicial effect; and the drug sale is admissible if its probative value outweighs its unfair prejudicial effect. 4. Both crimes are admissible if their probative value outweighs their prejudicial effect.

3. The false police report is admissible only if its probative value substantially outweighs its unfair prejudicial effect; and the drug sale is admissible if its probative value outweighs its unfair prejudicial effect. This is another question where you should approach each piece of evidence one by one. The first conviction is over 10 years old, so the reverse 403 standard, the most difficult one to meet applies. This makes sense - people change so we don't hold old crimes against them. The second conviction is less than 10 years old because he was released eight years ago. It is not a crime of dishonesty (drug sale), so you inquire whether it is the standard for a criminal defendant or any other witness. He is a criminal defendant so you use the standard that leans more toward exclusion than Rule 403 but less toward exclusion than the reverse 403 test. The middle standard - admit if probative value outweighs unfair prejudice to the defendant. This means answer 3 is most correct. (We don't have info about juvenile convictions and pardons so assume they do not apply.)

Todd was driving his van when a cow seemingly fell out of the sky and damaged his front hood. The day before, the cow was competing in the Prairie Fun League Livestock Exhibit. The judge wrote on the scorecard: "This cow seems unsteady on her feet." Will this be admissible under business record exception? 1. No, exception doesn't cover opinions. 2. No, hearsay within hearsay 3. Yes

3. Yes

A "good faith belief" under Rule 608(b) is most like ... 1. the prima facie standard 2. a preponderance of the evidence 3. probable cause for a warrant 4. clear and convincing evidence

3. probable cause for a warrant

Frieda is the manager of a small bank in Fairbanks. On the night of January 10th, a male individual posing as a U.P.S. deliveryman approached her house with a package. Once inside her house, he put on a mask, grabbed Frieda, tied her to a chair and raped her, then spent the night in the house, leaving her bound. In the early morning he untied her and forced her at gunpoint to drive him to the bank, unlock the doors, and de-activate the alarm. He then went inside and took all the money from the cash registers and fled on foot. Calvin was arrested two days later on an anonymous tip, and the police found large stashes of money in his house. He was picked out of a line-up by Frieda, who admits she only saw his face for the first few moments when he approached her house the night before. Calvin was charged with rape, kidnapping, and armed robbery. In the prosecution's case in chief, they wish to offer the following evidence: (1) The testimony of Ted, the bank manager of a small bank in Akron. Ted will testify that five years ago, Calvin posed as a U.P.S. deliveryman and thus gained entry to Ted's home. He tied Ted to a chair and left him bound all night, and the next morning forced Ted to drive him to Ted's bank, unlock the doors and disarm the alarm system. Calvin then stole all the money in the cash registers. (2) A certificate of conviction for rape, in which Calvin was convicted of raping a woman ten years ago in his house after they had come home from a date. What should the judge do? 1. Preclude both (1) and (2) 2. Admit (1) (subject to a rule 403 balancing test) and preclude (2) 3. Preclude (1) and admit (2) (subject to a rule 403 balancing test) 4. Admit both (1) and (2) (both subject to a Rule 403 balancing test)

4. Admit both (1) and (2) (both subject to a Rule 403 balancing test) For this question you should take up each piece of evidence one by one. Start with the second piece of evidence because it is easier to address. The rape will come in under Rule 413, the exception that permits prior sexual assaults to be used in sexual assault cases. This leaves you to choice between C & D. Then look at the prior robbery. It cannot be used to argue he is the type of person who robs so committed this robbery. It is a unique enough mode of robbery that it can be admitted, subject to Rule 403, for identity. This means D is the correct answer because both are admissible.

Tara is shot outside her dormitory by an unknown assailant. The next day while in the hospital, she is visited by Officer Snyder, who asks her questions about the shooting. Officer Snyder takes notes on the interview but does not show them to Tara. The officer then shows Tara ten pictures of possible suspects. Tara studies them carefully and then points to picture #4, saying: "He's the one who shot me, I'm sure." Picture #4 is a picture of Warren, who is immediately arrested and charged with attempted murder. At trial, Tara is unable to identify Warren as the assailant. She testifies that she remembers looking through the photographs at the hospital, but is unable to remember which one she chose. The prosecutor then asks Tara if seeing the officer's notes would refresh her recollection, and Tara says yes. The prosecutor asks the judge for permission to approach and provides opposing counsel with a copy of the notes. Tara looks at the notes, hands them back to the prosecutor, and then testifies that she selected picture #4 from the series of photos. The defense attorney objects and moves to strike. What should the judge do? 1. Preclude the testimony because Tara's statement identifying picture #4 is inadmissible hearsay. 2. Preclude the testimony because the police report cannot be used against Warren in this way. 3. Preclude the testimony for both reasons--the prior identification is inadmissible hearsay AND the police report cannot be used against Warren in this way. 4. Admit the testimony.

4. Admit the testimony. Here you have two problems. You should recognize and issue from the witness segment about whether the attorney has properly refreshed the witness' recollection and a hearsay issue because the identification was an out-of-court statement at the police station. Check your steps for refreshing recollection. The attorney followed the steps so there is not problem there. Look for a hearsay exception that will admit the statement. The first one we check is prior statements. Here the witness who made the identification, Tara, is on the stand under oath and subject to cross-examination. There are no further requirements for the prior statements exemption so her identification is admissible. The correct answer is 4.

Witnesses may refresh recollection from .... 1. Personal notes 2. Official reports 3. Any writing 4. Anything

4. Anything

On direct examination, leading questions are allowed ... 1. Never 2. When necessary to develop the witness's testimony 3. For a hostile witness or adverse party 4. Both 2 and 3

4. Both 2 and 3 Rule 611

•Widget sues Copy Company for patent infringement •Widget VP gives deposition under oath •Widget and Copy Company settle •VP dies •Widget sues Knock Off Company for infringement Widget offers VP's deposition testimony under 804(b)(1). Knock Off's best objection is .... 1. Other VPs are available 2. Depositions aren't "hearings" 3. Copy Co asked no questions at the deposition 4. Copy Co had different motives

4. Copy Co had different motives

Rosenberg and Summers are arrested for robbing a convenience store. The judge grants a mutual defense motion for severance and Rosenberg is tried first. At Rosenberg's trial, the prosecution calls Faith, the clerk at the convenience store, who was the only eyewitness to the crime. Faith testifies that a woman wearing a yellow jacket came into the store and put a gun to her head while a second woman wearing a denim jacket emptied the cash register. A police officer later testifies that Rosenberg and Summers were arrested minutes later one block from the store, and Rosenberg was wearing a yellow jacket and Summers was wearing a denim jacket. Rosenberg is convicted, and Faith, the eyewitness, dies shortly thereafter. At Summers' trial, the prosecutor seeks to admit the court transcript of Faith's testimony. The defense attorney objects. The evidence is: 1. Admissible under Rule 804(b)(1). (former testimony) 2. Admissible under Rule 801(d)(1). (prior statement) 3. Admissible under Rule 803(8). (public record) 4. Inadmissible

4. Inadmissible Again, once you look at the answer choices you know you are trying to find a hearsay exception or exemption that will permit admitting Faith's testimony at Rosenberg's trial for the truth of the matter asserted. You should be able to relatively easily rule out answer 3 because prior testimony is not a government agency's activities, findings, or report. You should also be able to relatively easily rule out answer 2 because for the prior statement exemption to apply the witness must be on the stand, and here Faith is dead so not on the stand. As to former testimony, this is a criminal case so the rule requires that the same parties have to be involved. The current defendant is Summers whereas the prior defendant was Rosenberg. Because they are not the same parties, this means the correct answer is that the prior testimony is inadmissible.

Vanessa is suing an upscale French restaurant owned by Marcel. Vanessa was dining with her friend Shrerekha at the restaurant when she cut her hand on a crystal water goblet that had a shard of glass sticking out from a prior breakage. The next day Shrerekha received a call from a woman who told her the following: "Your friend should sue Marcel because he knew that glass was broken and could injure a customer. I had told him the day before that he needed to remove that goblet from those in use because it was dangerous." If Shrerekha takes the stand to testify to her conversation with the caller then the statement that "the goblet was dangerous" is: 1. Admissible to prove only that the goblet was dangerous 2. Admissible to show only that Marcel knew the goblet was dangerous. 3. Admissible to show both that the goblet was dangerous and that Marcel knew the goblet was dangerous 4. Inadmissible

4. Inadmissible The key to answering this question correctly is to identify that there are two levels of hearsay. There is the callers statement to Marcel and then the callers statement to Shreerekha conveying her statement to Marcel. Both of these statements are made out of court. While the statement to Marcel would be admissible to prove knowledge but not that the goblet was dangerous had the caller taken the stand, the caller is not on the stand. Instead Shreerekha is on the stand, and Vanessa is trying to use the caller's statement to Shreerekha for the truth of the matter - that the caller had the described conversation with Marcel. No hearsay exception permits admitting that statement so no part of what was conveyed by the caller is admissible.

To object to the judge testifying, a party must 1. File a motion in limine 2. Object asap 3. File a written motion 4. Need not do any of the above

4. Need not do any of the above

Can a Prosecutor introduce evidence related to a civil settlement to show guilt? 1. Yes, but only the fact of settlement. No details. 2. Yes, if the crime is a felony 3. Yes, but subject to rule 403 4. No

4. No

Todd was driving his van when a cow seemingly fell out of the sky and damaged his front hood. The day before, the cow was competing in the Prairie Fun League Livestock Exhibit. The League President wrote a letter to Callie, the owner of the cow. It read: "Dear Callie, I saw one of your cows lurching about the pasture when I drove by today. She seems very unsteady, as if she might fall off a cliff. Sincerely, Elwood Pratt." Is Pratt's letter admissible as a business record? 1. Yes 2. Yes, if probative value substantially outweighs unfair prejudice 3. Yes, as long as it is trustworthy 4. No

4. No

•Government charges Joseph with insider trading •Joseph's wife testifies before grand jury •At trial, she claims spousal privilege and refuses to testify. Admit Wife's Grand Jury Testimony under 804(b)(1)? 1. Yes 2. No, she's still available 3. No, it wasn't a hearing 4. No, Joseph had no chance to cross-examine

4. No, Joseph had no chance to cross-examine

Meredith told her friend, Tess, about her new job. Meredith said, "Last week, I accepted a job with the U.S. Attorney's Office." Can Tess testify about Meredith's statement to show she had job with U.S. Attorney? 1. Yes, state of mind. 2. Yes, not hearsay. 3. Yes, excited utterance. 4. No, hearsay.

4. No, hearsay.

Katrina lives near wilderness, and wildfires have broken out. Katrina says, "I'm scared. The fires have been raging for weeks." What does the sentence "The Fires have been raging for weeks" fall under? 1. Excited utterance 2. Present sense impression 3. State of mind 4. None of these

4. None of these

U.S. v. Zlatogur •Zlatogurs charged with violating immigration laws •Yuri Yezhek, their employee, gave incriminating grand jury testimony •Yezhek left USA Is Yezhek's grand jury testimony admissible as .... 1. A prior inconsistent statement—801(d)(1)(A) 2. Former testimony—804(b)(1) 3. For impeachment only—613 4. None of these

4. None of these

Plaintiff: "My surgeon told me I should stay home from work for at least six months." 1. Admissible, state of mind 2. Admissible, present sense impression 3. Admissible, medical treatment 4. Not admissible, hearsay

4. Not admissible, hearsay

Last year, James purchased a car from Leonard's Bare-Bones Car Shoppe. Two months after the purchase, he discovered the car was not new as had been represented by Christine, one of Leonard's employees. James stopped payment on the car at that point, and Bare-Bones sued James for payment. Bare-Bones calls Leonard as its first witness. PLAINTIFF ATTORNEY: How are you employed? LEONARD: I work for Bare-Bones Car Shoppe. PLAINTIFF ATTORNEY: Are you in fact the owner of Bare- Bones Car Shoppe? DEFENSE ATTORNEY: Objection! The judge should: 1. Sustain the objection, since this is a leading question on direct examination. 2. Overrule the objection, since this is a non- leading question. 3. Overrule the objection, since the witness is an adverse party. 4. Overrule the objection, since leading questions are permissible for this type of question on direct examination.

4. Overrule the objection, since leading questions are permissible for this type of question on direct examination. Rule611(c). Leading Questions Leading questions should not be used on direct examination except as necessary to develop the witness's testimony.

Wendy, an avid Obama supporter, snuck into a campaign rally held by John McCain and shouted "Down with McCain! Vote for real change!" At that point someone standing next to Wendy punched her in the face and dislocated her jaw. Wendy accused Jack of the assault and sued him for battery in a civil suit. Jack is a loyal McCain supporter who was attending the campaign rally and standing near Wendy while she was heckling. At trial, Jack's attorney offers (1) a witness who will testify that at a McCain campaign event six months earlier, Jack was standing right next to a heckler from the Romney campaign who screamed obscenities at McCain for five minutes until security escorted her out, and that the whole time Jack simply smiled at the situation, and (2) a witness who would testify that he has known Jack for ten years and that in his opinion Jack is a peaceful man who never loses his temper. The court should: 1. Admit both pieces of evidence 2. Admit the evidence of Jack's actions at the prior McCain rally but preclude the opinion evidence of his character 3. Preclude the evidence of Jack's actions at the prior McCain rally but admit the opinion 4. Preclude both pieces of evidence

4. Preclude both pieces of evidence The relevance of both of these pieces of evidence depends on a propensity argument. Jack previously stood peacefully near a Clinton heckler, so Jack is a peaceful type of person, so Jack usually stays calm around hecklers, so Jack didn't punch Wendy. Jack is a peaceful man, he doesn't lose his temper, so he didn't punch Wendy. The evidence is not admissible under Rule 404.

Harry, a landlord, is being sued by his tenant Kathryn for failing to repair dangerous conditions in the apartment that he leases to her. Kathryn's attorney calls Harry to the stand and asks him: "Isn't it true that Kathryn sent you several letters complaining about conditions in the apartment?" Harry replies that he never received any such letters. Kathryn's attorney then approaches Harry with papers in her hand and asks whether seeing copies of the letters that Kathryn had sent him would refresh his recollection. Harry's attorney objects. How should the judge rule? 1. Overrule the objection and allow Kathryn's attorney to show Harry the letter to refresh his recollection. 2. Sustain the objection, since you cannot refresh a witness' recollection on cross-examination. 3. Sustain the objection, because the letters are not in evidence. 4. Sustain the objection, because Harry never said he had trouble remembering whether he received any letters.

4. Sustain the objection, because Harry never said he had trouble remembering whether he received any letters.

Barack files a libel suit against a company, "Chicago Citizens for Truth," which has run commercials in which members of the organization call Barack a "crook who took backdoor money from developers in exchange for votes." At trial, Chicago Citizens offers the following evidence: (1) testimony from a staffer who worked for Barack when he was a state legislator, who testifies that he was present in the room when developers gave Barack suitcases full of cash; and (2) testimony from a former college friend who testifies that he observed Barack smoking marijuana and drinking heavily while he was a state legislator. After this evidence is admitted, Barack's attorney offers (3) testimony from a developer in Chicago who states that he once offered to take Barack out to dinner to discuss a pending bill, and that Barack refused, saying that he couldn't accept any favors from private developers. Which (if any) of this testimony should be admitted? 1. The court should have precluded all three pieces of evidence under Rule 404 2. The court properly allowed in all three pieces of evidence 3. The court should have allowed in (1), Chicago Citizens' evidence of Barack taking suitcases full of cash, but should have precluded the other two pieces of evidence. 4. The court should have allowed in (1) and (3), but should have precluded the evidence of Barack's drinking and marijuana use

4. The court should have allowed in (1) and (3), but should have precluded the evidence of Barack's drinking and marijuana use The first key word to notice in this question is "libel." What do we know is usually at issue in libel suits? The character of the defamed person. So any pertinent evidence of Barack's character is admissible. Take each piece of evidence in turn. The first testimony is highly probative, so is admissible. The second testimony is not pertinent - whether someone smokes and drinks does not make it any more likely they took cash from developers, and if probative, the low probative value is likely outweighed by unfair prejudice. The third is pertinent and will be admitted. So answer 4 is the correct choice

Southern Ohio. 16 year old girl, Cherly Fossyl, disappeared in 1997. Body found several weeks later. Police did not solve crime. Some believed they participated in a cover up. In 2005, Jean Ann Chinn was dying of cancer in Florida. She told her brother she had witnessed the murder, and named 2 men. Chinn died two months later. Grand jury - not enough evidence for criminal prosecution. Fossyl's siblings filed civil suit against alleged murderers and county. County settled. 2007 trial. Can the defendants prove that Chinn had been convicted of assault? 1. No, she's not a witness 2. No, that's character evidence 3. No, it's not a crime of dishonesty 4. Yes, if it was a felony conviction satisfying Rule 609

4. Yes, if it was a felony conviction satisfying Rule 609

• A man beats Mamie brutally • As she collapses, Mamie says: "I'm dying! Roscoe did this; don't let him get away with this!" • Mamie survives, but has amnesia • Government prosecutes Roscoe for assault Can the neighbor testify about Mamie's statement? 1. No, it's hearsay 2. No, because Mamie is available 3. Yes, it's a dying declaration 4. Yes, it's an excited utterance

4. Yes, it's an excited utterance

•Sherry tells police: "My boyfriend is a gangster. He'll kill me if he finds out I talked to you." •Gives detailed report about boyfriend's crimes •Sherry disappears Is Sherry's report admissible against boyfriend at trial? 1. Yes, statement against interest 2. Yes, excited utterance 3. Yes, state of mind 4. Yes, dying declaration 5. No

5. No

Among other reasons, the Federal Rules of Evidence restrict the types of evidence admitted 1. To ensure evidence is sufficiently reliable 2. To ensure absolutely no delay is tolerated 3. To protect juries from misleading information 4. All of the above 5. One and three

5. One and three The rules do limit delay, but they cannot absolutely eliminate all delay.

What is a character witness?

A character witness testifies about the untruthful or truthful character of a fact witness

Polly sues Don for an assault that occurred on May 8 in Arizona. Don claims he was in Utah on May 8. Don offers a letter he wrote to his sister on February 12: "See you in Utah on May 8." Admit Don's letter to support claim that he was in Utah? A. Admissible, state of mind. B. Admissible, excited utterance. C. Irrelevant D. Inadmissible speculation

A. Admissible, state of mind.

Francis was living on board his Yacht "Charity" when it exploded one morning, killing him instantly. Investigators determine that the cause of the accident was propane from the stove in the boat that escaped and filled the cabin, causing an explosion when Francis tried to light the stove in the morning. Francis' estate sues the manufacturer of the stove, claiming that it developed a leak which caused the gas to escape. The defendant manufacturer has some (inconclusive) physical evidence recovered by divers that the gas for the stove might have been negligently left on the night before, causing gas to escape into the boat, and the defendant claims that Francis was at fault for not turning off the gas after he was done cooking the night before. At trial, the attorney for Francis' estate calls Samantha, who has been on the boat with Francis at least twenty nights over the past year, though she was not with him on the night before the explosion. She testifies that every night before going to bed, Francis would always take five minutes to check the anchor, the sail ties, the motor lock, and the cooking gas. The defendant manufacturer objects. The evidence should be: A. Admitted under Rule 406 as a habit B. Produced under Rule 411 as liability insurance. C. Precluded under 404(a) as propensity evidence. D. Precluded under Rule 403.

A. Admitted under Rule 406 as a habit We didn't study Rule 411 so you can knock answer B out right away. There also doesn't seem to be anything unfair about this evidence so you can eliminate D. That leaves you to decide whether the evidence is propensity evidence or non-propensity evidence demonstrating a habit. Ask whether the conduct is repeated, specific, and in response to a distinctive situation. Yes, it is repeated every night of nights Francis has been there, it occurs before bed, and it is the same type of safety check. So the correct answer is A.

What is hearsay?

An out-of-court statement offered to prove the truth of the matter asserted.

John Grisham, a famous author, is accused of killing Xavier Daniels. When Grisham was arrested, the police found a letter in his coat pocket written by Daniels and addressed to Grisham. The letter said: "You plagiarized almost all of your last novel from my father's work. I know it. I demand that you pay me $500,000, or I will sue you in court and destroy your reputation." The prosecutor wishes to admit the letter as evidence against Grisham. The letter is: A. Hearsay if used to prove motive. B. Hearsay if used to prove that Grisham plagiarized his last novel. C. Hearsay for either of the two purposes. D. Not Hearsay no matter what it is meant to prove.

B. Hearsay if used to prove that Grisham plagiarized his last novel. This time when you read the answers, you know for certain you are dealing with a hearsay issue. Is the letter an out-of-court statement? Yes. Is the letter a statement? Yes, Daniels is communicating that Grisham plagiarized his last novel. What is the truth of the matter? The prosecutor cannot use the statement to prove that Grisham plagiarized his last novel, that is the truth of the matter asserted in the letter. Looking at the choices that means b is correct. Yet, even if the statement were not true, Grisham might still have a motive to kill Daniels because he didn't want his reputation destroyed. The use of the statement to prove motive is a non-hearsay purpose - it doesn't depend on whether the statement's content is true or false. Because of this you know that c is not correct.

Direct Evidence

evidence that shows a fact

Brett Hankison, a former police officer, is being criminally prosecuted for wanton endangerment for shooting a firearm without being cognizant of his target and shooting in a reckless manner into a neighboring apartment. The prosecution must prove a wanton state of mind to prevail. The prosecution filed a motion in limine to introduce evidence of a prior incident in which Hankison was found to have driven recklessly and hit another police officer in order to prove Hankison's wanton state of mind. Regarding that incident, the Professional Standards Unit had concluded Hankison had failed to drive with due regard for the safety of all people. Under the Federal Rules of Evidence a judge would: A. Admit the prior incident to prove that Hankison is a reckless person so probably discharged his firearm in a reckless wanton manner. B. Admit the prior incident to prove that Hankinson has the propensity to act in a reckless manner. C. Admit the prior incident only to prove intent but not to prove propensity and only if its probative value is not substantially outweighed by unfair prejudice. D. Admit the prior incident to prove that Hankinson has the propensity to act in a reckless manner only if a limiting instruction is requested.

C. Admit the prior incident only to prove intent but not to prove propensity and only if its probative value is not substantially outweighed by unfair prejudice. Rule 404 prohibits propensity evidence. Choices 1, 2, and 4 all suggest the judge would admit the prior incident precisely to use for prohibited propensity purposes -- that is the jury could reason that because Hankison was reckless during another incident, he is a reckless person, and therefore he was reckless when using his firearm during the incident at issue. There is no exception that would permit using the incident in this way. We are not dealing with a crime of dishonesy. The evidence is not being offered by the defendant, and this is not a homicide case. The incident is not one of sexual assault or child molestation. This leaves 3 as the only correct answer. And, indeed, this answer suggests the incident could only be admitted for a non-propensity purpose, intent, and only if it passes muster under Rule 403.

Late one night, Officer Reynoso and Officer Girardi noticed a suspicious person loitering outside a convenience store. They approached the individual to ask some questions, and Officer Reynoso heard Girardi yell: "Look out, he's pulled out a gun!" Reynoso then saw Girardi pull out his gun and begin shooting at the individual. Reynoso immediately drew her own gun and shot six rounds at the individual. The individual was struck by a total of eleven bullets and was killed instantly. A firearm was found near the victim's body, but there is some evidence that it might have been planted there after the shooting took place. The two police officers were charged with second-degree murder following the incident. Their cases were severed, and Officer Reynoso is now on trial. Her defense is that she reasonably believed that the victim had a gun and thus the shooting was justified. In her defense, Reynoso wants to testify as to (1) the statement "Look out, he's pulled out a gun!" made by Girardi, and (2) the fact that Girardi drew his own gun and began shooting it. The prosecutor objects to both pieces of evidence on hearsay grounds. What is the proper ruling? A. Both the statement by Girardi and Girardi's drawing of his gun are inadmissible for any purpose. B. The statement by Girardi is inadmissible for any purpose, and Girardi's drawing of his gun is admissible only to prove Reynoso reasonably believed the victim had a gun. C. Both the statement by Girardi and Girardi's drawing of his gun are admissible for any purpose. D. Both the statement by Girardi and Girardi's drawing of the gun are admissible to prove Reynoso reasonably believed the victim had a gun, but cannot be used as evidence that the victim actually had a gun.

C. Both the statement by Girardi and Girardi's drawing of his gun are admissible for any purpose. Take each piece of evidence in turn. The statement doesn't seem to pose any propensity issues or other issues related to witnesses or character. It is a statement which is a red flag that there may be a hearsay issue. It was made out of court and is an assertion. The truth of the matter asserted is that the victim has a gun. Reynoso isn't necessarily using the statement to prove the victim had the gun, but only to show why she believed the victim had a gun. So d may be correct. You can rule out a & b. But you need to move to the next step and see if any exception will admit the statement for the truth of the matter. You can safely assume it is an excited utterance because it begins with "Look out" which is an exclamation and is in a tense situation. This means c must be the correct choice. You can make sure or begin with the second piece of evidence which is Reynoso saw Girardi draw a gun and start shooting. This is conduct out in the world that Reynoso observed. It doesn't pose any witness, propensity, character, or hearsay issue. That means you could eliminate a, b, and d.

John is on trial for raping Cynthia. Cynthia claims that they met in a bar, and after an hour he offered to give her a ride home. She claims that when she got into his car he drove her to a secluded location and raped her in the car. John's defense is consent. He claims that they met in a bar, and after an hour Cynthia said: "Want to take me for a ride?" When John agreed, they went to his car and she directed him to the secluded location, saying it was "my favorite spot; nobody around for miles." They parked and had consensual sex. At trial, John wishes to introduce the following evidence: (1) Three different men will testify that on separate prior occasions over the past month, Cynthia has met each of them in the same bar and after some conversation asked "Do you want to take me for a ride?", then directed them to the same secluded area, describing it as her "favorite spot, nobody around for miles" and had sex with them. (2) A witness will testify that ten minutes before John and Cynthia left the bar together in John's car, he saw them in a back room in the bar "kissing passionately and feeling each other up." (3) Testimony by the bartender at the bar that he is familiar with Cynthia's reputation, that she is present at the bar almost every night, and that she is known as "easy," and that "everyone knows she always goes all the way on the first date." Which of these (if any) are admissible? A. (1), (2), and (3) B. (1) and (2), but not (3) C. Only (2) D. None of them

C. Only (2) The starting point is that Rule 412 the rape shield would prohibit each of these pieces of evidence because they deal with sexual acts or disposition. Check each to see if they fit any exception to the prohibition. Only the second piece of evidence fits in an exception. It is a prior act with the defendant who is pleading consent that is admissible under 412(b)(1)(B).

Ed is riding in a rodeo when his horse steps into a huge gopher hole in the southwest corner of the arena, causing the horse to fall and throwing Ed into the fence of the arena. Ed breaks his leg and sues Andy, the owner of the arena, for negligently maintaining the grounds. During the trial, Ed calls Jack as a witness. Jack works at the arena and will testify that the day before the incident, he overheard a conversation between Ron, another rodeo rider, and defendant Andy. Jack testifies that during this conversation Ron told Andy: "There's a large hole near the southwest corner; my horse almost stepped into it." Andy's attorney objects to Jack's testimony, claiming the statement by Ron is hearsay. Which of the following is correct? A. The statement should be entirely precluded as hearsay. B. The statement should be admitted with no limiting instruction. C. The statement should be admitted with a limiting instruction that the statement should only be considered for the purposes of proving Andy knew there was a hole (assuming that is a relevant fact), not to prove there was in fact a hole. D. The statement should be admitted with a limiting instruction that the statement should only be considered for the purpose of proving the existence of the hole and for no other purpose.

C. The statement should be admitted with a limiting instruction that the statement should only be considered for the purposes of proving Andy knew there was a hole (assuming that is a relevant fact), not to prove there was in fact a hole. You know from the question this is a hearsay issue. Proceed through your steps to see if it could be used for a hearsay purpose. First, Ron made the statement out of court. Second, it was a statement - he was communicating to Andy that there is a large hole. This means if Ed's attorney wants to prove there was a large hole, then he would be using the statement to prove the truth of the matter. That is prohibited, so he will need other evidence that there was a hole. At this point you can eliminate the last answer. Looking at the other answers you will see that c offers a non-hearsay purpose - using Ron's statement not to prove there was a hole but to prove Andy had notice that there was a hole. This means c is the correct answer. Why is the limiting instruction needed? Because the jury shouldn't consider Ron's statement as evidence there was a hole, they should only consider it to determine if Andy had notice of the hole.

True or False Dying declarations violate the 6th amendment

False

•Frieda is the manager of a small bank in Fairbanks. On the night of January 10th, a male individual posing as a U.P.S. deliveryman approached her house with a package. Once inside her house, he put on a mask, grabbed Frieda, tied her to a chair and raped her, then spent the night in the house, leaving her bound. In the early morning he untied her and forced her at gunpoint to drive him to the bank, unlock the doors, and de-activate the alarm. He then went inside and took all the money from the cash registers and fled on foot. Calvin was arrested two days later on an anonymous tip, and the police found large stashes of money in his house. He was picked out of a line-up by Frieda, who admits she only saw his face for the first few moments when he approached her house the night before. Calvin was charged with rape, kidnapping, and armed robbery. Calvin's attorney will argue, among other things, that it was not him. In the prosecution's case in chief, they wish to offer the following evidence: (1) The testimony of Ted, the bank manager of a small bank in Akron. Ted will testify that five years ago, Calvin posed as a U.P.S. deliveryman and thus gained entry to Ted's home. He tied Ted to a chair and left him bound all night, and the next morning forced Ted to drive him to Ted's bank, unlock the doors and disarm the alarm system. Calvin then stole all the money in the cash registers. (2) A certificate of conviction for rape, in which Calvin was convicted of raping a woman ten years ago in his house after they had come home from a date. What should the judge do? A. Preclude both (1) and (2) B. Admit (1) (subject to a Rule 403 balancing test) and preclude (2) C. Preclude (1) and admit (2) (subject to a Rule 403 balancing test) D. Admit both (1) and (2) (both subject to a Rule 403 balancing test)

D. Admit both (1) and (2) (both subject to a Rule 403 balancing test)

Gary is on trial in the Southern District Court of Florida for shooting and killing Francis as part of a contract killing. The killing occurred onboard a private yacht off the coast of Florida. Gary's primary defense is that he did not kill Francis until the yacht had reached international waters, and so United States courts have no jurisdiction over his actions. In his defense, Gary calls Ernie, who testifies that he was on board the yacht on the night of the killing and he heard the captain of the yacht tell the pilot: "Blow the horn three times when we reach international waters." A few minutes later Ernie heard the boat horn blow three times, and a moment later Ernie heard a gunshot. The prosecution objects to Ernie's testimony as hearsay. What is the correct ruling? A. The statement by the captain and the blowing of the horn are both hearsay. B. Neither the statement by the captain nor the blowing of the horn are hearsay. C. The statement by the captain is hearsay, but the blowing of the horn is not hearsay. D. The statement by the captain is not hearsay, but the blowing of the horn is hearsay.

D. The statement by the captain is not hearsay, but the blowing of the horn is hearsay. Once you have read the answer choices, you know you are dealing with a hearsay question, which understandably may not have been obvious just by reading the prompt. In this question we have to take each potential hearsay statement in turn. Is the captain's "statement" made out-of-court? Yes. Is it a "statement'"? No. Like a question generally the truth of a direction is not ascertainable. In other words, while it is a human communication it does not include an assertion. It simply directs another to do something. Like when I tell you "state the hearsay rule out loud." That can't be true or false, and you can follow my direction or not. Because of this you can eliminate choices a and c. Then turn to the pilot's blowing of the horn. Our first reaction should be that performing an action, blowing a horn, is not hearsay, but we need to investigate further and look for an implicit assertion as we did with the bank teller hitting the panic button. Is the blowing done out of court? Yes. Is it a statement? The pilot does the blowing. The horn doesn't blow by itself. So we have a human. The blowing communicates something to someone else. It says to the captain "We have reached international waters." That is an assertion. It is capable of ascertaining whether it is true or false. So we do have a implicit statement in the conduct. The blowing of the horn cannot be used to prove the truth of the statement - that the boat had reached international waters. The defense is arguing the boat had reached international waters which is precisely the same as the truth of the statement. This is hearsay, or using the statement for a hearsay purpose, and the blowing of the horn is not admissible for this purpose. This means d is the correct choice. As an aside in case you are wondering, since the blowing of the horn cannot come in, the captain's question will not provide relevant context and will likely not end up being testified to either. They need to call the pilot to testify when they reached international waters or find a machine that automatically registered that happening, and produce those records.

Brady was working outside in his yard when he saw a car pull up the driveway to his house. A man stepped out of the car carrying a shotgun and fired two blasts of the gun at Brady. The man then re-entered the car and drove away quickly. Brady's wife heard the shots and heard her husband yell "Help! Davis has shot me!" She came out to find Brady lying in the yard bleeding from the leg. He is taken to the hospital and two hours later is being prepped for surgery when the doctor tells him: "Do you know who shot you?" Brady replies in a calm tone of voice: "Yes. I am certain it was Davis." Davis is arrested and charged with attempted murder. Brady is unavailable to testify at trial, so the prosecutor calls the wife and the doctor to testify as to Brady's statements to them on the day of the shooting. What are the proper rulings? A. Both statements are admissible. B. Neither statement is admissible. C. The statement to the doctor is admissible, but the statement to the wife is not. D. The statement to the wife admissible, but the statement to the doctor is not.

D. The statement to the wife admissible, but the statement to the doctor is not. When we see that it is statements made by Brady out-of-court that are issue, we should realize we may have a hearsay issue. We take each statement in turn. First Brady's statement to his wife. He asserts that Davis shot him, and the prosecutor wants to offer the statement to prove it was Davis who shot Brady. So we look for a hearsay exception. At the time of making the statement Brady has just been shot and screams help, which indicates this is an excited utterance that will be admissible under 803(1). You can go through your checklist of exceptions to be sure. It isn't a prior statement because Brady isn't on the stand. It could arguably be a present sense impression because it happens immediately after the event, and arguably is descriptive, although he reaches a conclusion about who the shooter is. It isn't a state of mind. Next look at the second statement. Again it asserts Davis shot Brady and that is what the prosecutor is using the statement to prove. Go through your list of exceptions. Again it isn't a prior statement or a state of mind. Now neither excited utterance or present sense impression will work to get this statement in because Brady is calm at the time and is not describing a event as it happens or immediately thereafter. This means the correct choice is answer d. Also a friendly reminder that you can start with either statement you are more certain about and eliminate choices that don't fit your conclusion as to the statement if that is helpful

Excited Utterance

Declarant made statement while excited by the event. Statement related to the event.

Admit or Exclude: Defendant wants to introduce evidence that alleged rape victim, Sally, is on birth control pills.

Exclude

Habit or Not: Defendant has coffee at the diner every Sunday morning at 10 am

Habit

Immediately after hearing Mark announce his severe peanut allergy, Josh smeared peanut butter across Mark's forehead. Josh is charged with assault. If offered to show that Josh smeared PB on Mark, is the following statement made by the Cafeteria Worker hearsay or not hearsay? "I did not see the incident, but I talked to both Mark and the student sitting next to him in the cafeteria. Both confirmed that Joshua Hickson smeared peanut butter on Mark's forehead."

Hearsay

The Sixth Amendment In all ___________ proceedings, the ___________ shall enjoy the right to be ______________ with the _______________ against him.

In all criminal prosecutions, the accused shall enjoy the right to be confronted with the witnesses against him

Hal and Falstaff got into a fight in a bar, and Hal pushed Falstaff to the ground and repeatedly kicked him in the head. Hal was arrested for assault, and Falstaff sued Hal for battery. One week before the criminal trial, Hal paid Falstaff $10,000 and Falstaff dropped the battery case against him. During the criminal trial, the prosecutor calls Falstaff to the stand and asks him whether Hal paid money to him after the assault. The defense attorney objects. Falstaff's answer would be: 1. Inadmissible to prove Hal's guilt because it is irrelevant 2. Inadmissible to prove Hal's guilt under rule 408 2. Inadmissible to prove Hal's guilt under rule 410 4. Admissible to prove Hal's guilt

Inadmissible to prove Hal's guilt under Rule 408, settlement negotiations. The rule prohibits use of settlement negotiation to prove guilt. Note that there was a civil claim for battery which the parties settled, making the negotiations a settlement covered by the Rule.

Present Sense Impression

Made while perceiving the event or immediately thereafter. Description of event.

Hearsay or Not: A witness testifies about the "X" on the back of Daniel's hand is to show that he is under 21. Admit?

No, it's hearsay

Is the bark of a K-9 hearsay?

No, not a statement made by a human

Are non-verbal behavior actions by humans normally statements?

No, so hearsay is not an issue.

Would you introduce evidence from a psychic who says that due to the Defendant's aura, he was definitely not drinking and driving?

No. Culturally our society doesn't accept that and the jury would roll their eyes.

Can Betsey testify that she was a virgin? (Evidence: Complainant's Virginity)

No. This evidence would be excluded.

Is evidence that a Defendant accused of selling coke used weed relevant?

No. Those are very different things. Using drugs does not equate to selling drugs.

Habit or Not: I have tripped and spilled a beverage 6 times in 2 months.

Not habit

When is evidence of truthful character admissible?

Only after the witness's character for truthfulness has been attacked.

Henry is suing Susan in a wrongful death action for Henry's wife Florence. Henry alleges that Florence was a passenger in Susan's car, and that Susan was driving 90 miles an hour down an icy road, lost control of the car, and hit the side of an overpass. This collision caused Florence to fly through the windshield, resulting in fatal injuries. Henry offers into evidence several photographs which show Susan's car with a mangled front end and graphic bloodstains on the inside of the windshield. Henry claims the extensive damage to Susan's car tends to prove how quickly Susan was driving, and that the bloodstains and the hole in the windshield help to prove cause of death. Susan's attorney objects under Rule 403. She states that she is willing to stipulate that Susan was traveling at 90 miles per hour, and also to the fact that Florence was killed by being thrown through the windshield as a result of the collision. How do you rule?

Overrule the objection, since the probative value of the photos are not substantially outweighed by their unfair prejudice.

Tom and Jerry got into a fight at a local bar, and Jerry ended up beating Tom so badly that Tom was taken to a hospital. Jerry is prosecuted for assault. At trial, he admits to the assault but claims he was acting in self-defense and that Tom started the fight after having drunk too many beers. In his defense, Jerry wishes to introduce evidence that on three prior occasions in the past week, Tom had drunk too much in the bar and picked a fight with another patron. The prosecution objects, pointing out that Jerry had not been present for any of these prior occurrences and had no knowledge of Tom's propensity for violence at the time of the alleged assault. The evidence of Tom's three prior fights is:

Relevant, since a person who tends to get violent when he drinks is more likely to start a fight than a person who does not.

Statement or Not: Medical form: "List any lung problems." Patient leaves blank. Has written response to every other question.

Statement

Todd was driving his van when a cow seemingly fell out of the sky and damaged his front hood. The day before, the cow was competing in the Prairie Fun League Livestock Exhibit. The judge wrote on the scorecard: "Michelle, the cow, weighs 600 pounds." Will this be admitted as a business record?

Yes

Maria Young sued her boss and company for sexual harassment. She calls a coworker to testify. Coworker will testify that she repeatedly saw Maria crying and looking stressed or unhappy after meetings with the boss. The boss objects under Rule 602, noting that the coworker never witnessed any harassing interactions between Maria and the boss. Can the coworker testify about Maria crying?

Yes The co-worker saw and heard (sensed) Maria crying. She has knowledge of this relevant circumstantial evidence. Someone who cries after meeting with the boss is somewhat more likely to be upset because the boss is harassing them. Evidence does not have to be direct (eyewitness testimony) to be admissible.

Are questions about specific instances permitted on cross-examination?

Yes! Because the oath will make the jury believe the character witness's opinion unless it can be undermined on cross.

Admissible as habit? Professor Levinson fills the thermos with coffee ever day before class.

Yes, habit

Admissible as habit? Professor Levinson brings her silver thermos to class everyday.

Yes, it's a habit

Mark charged with raping Cindy. Mark denies any sex with Cindy. Semen found on Cindy's underwear, but DNA testing was inconclusive. Can Mark introduce evidence that Cindy had sex with her boyfriend, Jules, the day of the alleged rape?

Yes, that evidence can be admitted.

Is an "X" on the back of a bar attendees' hand a statement?

Yes, that is a statement.

Are photos of a dismembered victim relevant, even if the Defendant agrees to stipulate the manner of the deceased?

Yes. The government does not have to stipulate even if the Defendant wants to stipulate. Just because something is gory, it is rarely prejudicial to the Defendant.

•Max and Diane are charged with attempting to illegally purchase semi-automatic machine guns with the intent of reselling them to a local street gang. Max was caught red-handed in a sting: He gave $5,000 to an undercover ATF agent after arranging to buy firearms from the agent. Diane was not present at that time. •Max pleads guilty, but Diane goes to trial. Which of the following statements would be admissible against Diane, assuming there is corroborating evidence of the conspiracy between Max & Diane? (select all correct answers) a. A recording, made via wiretap, of Max speaking to Diane on the phone. Max says: "I've set up the meeting with the seller at 3:00 Saturday; I'll meet you with the buyer at 4:00 at the warehouse." b. Testimony by the undercover agent that on the day before the attempted purchase, Max said to him: "Meet me at 3:00 tomorrow; we have to get this done quickly because I need to meet up with my partner Diane and the buyers an hour later." c. Testimony from one of the arresting police officers that after Max was arrested, he said: "It's true I was trying to buy the guns, but Diane was the real ringleader of the operation." d. Testimony from a friend of Max that two days before the attempted purchase, Max said: "This lady Diane and I are working on a sweet deal that should help me make some quick money." e. Testimony from a friend of Diane that two days before the attempted purchase, Diane said: "I invested my savings in a business arrangement with this guy Max, and I should be able to double my money in just a few days."

a. Yes, as co-conspirator statement. b. yes, as a co-conspirator statement e. yes, as opposing party statement

The Columbus Dispatch published a front-page story calling the lottery commissioner of Ohio "a thief who steals money from the public purse for his own personal gain." The commissioner sues the Dispatch for libel. In its defense, the Dispatch wishes to offer evidence that: (1) the commissioner diverted funds from the Ohio state lottery to pay for renovations to his home; (2) five years ago the commissioner was working for the Indiana state lottery and used funds from that lottery to buy a new car; and (3) last year the commissioner was arrested for drunk driving but used his connections in state government to cover it up. What are the proper rulings for these pieces of evidence? a. (1), (2), and (3) are all admissible. b. (1) and (2) are admissible, but not (3). c. (1) is admissible, but not (2) or (3). d. (1), (2), and (3) are all inadmissible.

b. (1) and (2) are admissible, but not (3). The key word comes later in this question but there it is again "libel." So we know character is at issue. Is he a thief or not? Take each piece of evidence in turn. The first is pertinent, and the evidence is admissible; the second is pertinent, and is admissible. The third, however is for drunk driving not for stealing - it doesn't bear on whether he is a thief, so it is not pertinent, and would not be admitted. The proper choice is b

A co-conspirator statement is made ...

during and in furtherance of the conspiracy

Under the FRE, an attorney representing a party at trial is incompetent to testify in that trial. true or false

false The Rules of Evidence do not prohibit an attorney from testifying. It isn't good practice though, and the Professional Responsibility Rules (PR) discourage it.

Counsel must have a _______ _______ _______ that a prior act occurred in order to raise it cross.

good faith belief

When may the court admit evidence of subsequent remedial measures?

impeachment or — if disputed — proving ownership, control, or the feasibility of precautionary measures.

Use Rule 407 to prove ownership/control, but can't use the rule to prove __________.

liability

motion in limine

motion to exclude or include evidence

Collateral evidence is evidence that is ...

not highly probative

A statement of ______ is admissible as state of mind and is often used as circumstantial evidence that those mentioned carried out the plan.

plan

A judge does not have to exclude evidence that is unfairly prejudicial. But why do we want to shield the jury from unfairly prejudicial evidence?

prevent unwanted bias leads jury to make irrational assumptions inflames passions (so the case is decided on grounds other than legal requirements)

Making a propensity argument, that someone has a certain character with which they acted or are acting consistently on a particular occasion, is generally ___________________ by the Rules of Evidence.

prohibited

Pertinent ______________ evidence is permitted in sexual assault and child molestation cases and may be introduced by any method.

propensity

When you (the attorney) have a record created by a government agency, you must use the ___________ ________ exception and not the business records exception.

public records

Why restrict evidence?

shield the jury efficiency societal interests reliability

On direct examination, an attorney can only ask a character witness for honesty or dishonesty and not about ___________ _______ of dishonesty or honesty.

specific acts

Habit is ...

specific conduct in a distinctive situation on a regular basis.

A statement of memory or belief is admissible as ________ ___ _______ as to remembering or believing but not for the truth of the matter as to what was remembered or believed.

state of mind

If the Defendant introduces evidence of their own good character evidence, then the Prosecutor can introduce evidence of ...

the defendant's bad character for the same character trait.

Who is not competent to testify under the Federal Rules? 1. Convicted felons 2. The judge 3. Catholics 4. Children younger than age six

the judge

If the Defendant introduces evidence of the victim's bad character, then the Prosecutor can introduce evidence of ...

the victim's good character for the same character trait AND evidence of the defendant's bad character for the same character

If the Defendant introduces evidence that the victim in a homicide case was the first aggressor, then the Prosecution can introduce evidence of ...

the victim's peaceful character

What purpose does rule 613 allow the attorney to use the inconsistent statement for?

to attack the credibility of the witness (purpose is not to prove anything, just to undermine the witness's credibility.)


Kaugnay na mga set ng pag-aaral

日本語総まとめN2第4週の語彙

View Set

Biology Photosynthesis definitions

View Set

MRU4.4: Supply and Demand Terminology

View Set

ECON 201 CH.4-7 review Multiple choice

View Set

chapter five: integumentary system check your understanding

View Set

Estructura 1.1 - 1 - ¿Masculino o femenino?

View Set

Quiz 20: Instructions and Procedures

View Set